Continuity of Linear Operator Between Hilbert SpacesNorm of adjoint operator in Hilbert spaceSesquilinear forms on Hilbert spacesGradient of inner product in Hilbert spaceDissipativity for Hilbert spacesA self-adjoint operator on a Hilbert spaceComplementary slackness in Hilbert spacesProof that every bounded linear operator between hilbert spaces has an adjoint.Proof explanation related to the operator matricesShowing that $exists x in H : |A(x)| = |A|_mathcalL$ if $H$ is Hilbert and $A in mathcalL_c(X,Y)$.Why is this operator symmetric? A question concerning a paper from Brezis and Crandall

Are all passive ability checks floors for active ability checks?

Knife as defense against stray dogs

Does someone need to be connected to my network to sniff HTTP requests?

Why doesn't the EU now just force the UK to choose between referendum and no-deal?

Creature kill and resurrect effects on the stack interaction?

What options are left, if Britain cannot decide?

Why one should not leave fingerprints on bulbs and plugs?

Could the Saturn V actually have launched astronauts around Venus?

If the DM rolls initiative once for a group of monsters, how do end-of-turn effects work?

Existence of subset with given Hausdorff dimension

How to create the Curved texte?

Recruiter wants very extensive technical details about all of my previous work

Is this a real picture of Jordan Peterson in New Zealand with a fan wearing a shirt that says "I'm a Proud Islamaphobe"?

Official degrees of earth’s rotation per day

Is a party consisting of only a bard, a cleric, and a warlock functional long-term?

Credit cards used everywhere in Singapore or Malaysia?

The difference between「N分で」and「後N分で」

If I can solve Sudoku can I solve Travelling Salesman Problem(TSP)? If yes, how?

It's a yearly task, alright

Define, (actually define) the "stability" and "energy" of a compound

Is it normal that my co-workers at a fitness company criticize my food choices?

Why is the President allowed to veto a cancellation of emergency powers?

If curse and magic is two sides of the same coin, why the former is forbidden?

Are ETF trackers fundamentally better than individual stocks?



Continuity of Linear Operator Between Hilbert Spaces


Norm of adjoint operator in Hilbert spaceSesquilinear forms on Hilbert spacesGradient of inner product in Hilbert spaceDissipativity for Hilbert spacesA self-adjoint operator on a Hilbert spaceComplementary slackness in Hilbert spacesProof that every bounded linear operator between hilbert spaces has an adjoint.Proof explanation related to the operator matricesShowing that $exists x in H : |A(x)| = |A|_mathcalL$ if $H$ is Hilbert and $A in mathcalL_c(X,Y)$.Why is this operator symmetric? A question concerning a paper from Brezis and Crandall













2












$begingroup$



Note: Please do not give a solution; I am curious to understand why my solution is incorrect, and would prefer guidance to help me complete the question myself. Thank you.





Let $mathcalH$ be a Hilbert space, and suppose that $TintextHom(mathcalH,mathcalH)$. Suppose that there exists an operator $tildeT:mathcalHrightarrowmathcalH$ such that,
beginalign
langle Tx,yrangle =langle x,tildeTyrangle,
endalign

$forall x,yinmathcalH$. Show that $T$ is continuous.



My current solution is as follows:



Assume for all $delta>0$ there exists $n>NinmathbbN$ such that,
beginalign
|x_n-x|<delta.
endalign

Then,
beginalign
langle Tx_n-Tx,Tx_n-Txrangle &= |Tx_n-Tx|^2\
&leq|Tx_n-Tx|=|T(x_n-x)|\
&leq|T||x_n-x|rightarrow 0text as nrightarrowinfty.
endalign



What am I doing wrong? I notice I do not use the existence of $tildeT$.



Second Attempt:



Assume $langle x_n,xrangle rightarrow langle x,xrangle$ as $nrightarrowinfty$. Then, given $langle Tx,yrangle = langle x,tildeTyrangle$,
beginalign
langle Tx_n,yrangle &= langle x_n,tildeTyranglerightarrow_nrightarrowinftylangle x,tildeTyrangle=langle Tx,yrangle.
endalign

Therefore, $Tx_nrightarrow Tx$ as $nrightarrowinfty$.



Third Attempt:



Assume $|x_n-x|rightarrow 0$ as $nrightarrowinfty$. Then,
beginalign
langle Tx_n-Tx,Tx_n-Txrangle=langle x_n-x,x_n-xrangle=|x_n-x|^2.
endalign



By assumption $|x_n-x|^2rightarrow 0$ as $nrightarrowinfty$. Hence,
beginalign
langle Tx_n-Tx,Tx_n-Txrangle = |Tx_n-Tx|^2rightarrow 0text as nrightarrowinfty.
endalign

Therefore, $T$ is continuous.










share|cite|improve this question











$endgroup$







  • 2




    $begingroup$
    The last inequality basically implies that the norm of T is bounded or that it is continuous
    $endgroup$
    – Andres Mejia
    4 hours ago






  • 1




    $begingroup$
    Comment on the second attempt: you showed that $Tx_n to Tx$ weakly, not in norm. Off-topic comment: I admire your tenacity. Keep trying!
    $endgroup$
    – Umberto P.
    4 hours ago











  • $begingroup$
    Thank you. Do you have a hint?
    $endgroup$
    – Jack
    4 hours ago










  • $begingroup$
    Third attempt made. Although not sure if this holds either.
    $endgroup$
    – Jack
    3 hours ago















2












$begingroup$



Note: Please do not give a solution; I am curious to understand why my solution is incorrect, and would prefer guidance to help me complete the question myself. Thank you.





Let $mathcalH$ be a Hilbert space, and suppose that $TintextHom(mathcalH,mathcalH)$. Suppose that there exists an operator $tildeT:mathcalHrightarrowmathcalH$ such that,
beginalign
langle Tx,yrangle =langle x,tildeTyrangle,
endalign

$forall x,yinmathcalH$. Show that $T$ is continuous.



My current solution is as follows:



Assume for all $delta>0$ there exists $n>NinmathbbN$ such that,
beginalign
|x_n-x|<delta.
endalign

Then,
beginalign
langle Tx_n-Tx,Tx_n-Txrangle &= |Tx_n-Tx|^2\
&leq|Tx_n-Tx|=|T(x_n-x)|\
&leq|T||x_n-x|rightarrow 0text as nrightarrowinfty.
endalign



What am I doing wrong? I notice I do not use the existence of $tildeT$.



Second Attempt:



Assume $langle x_n,xrangle rightarrow langle x,xrangle$ as $nrightarrowinfty$. Then, given $langle Tx,yrangle = langle x,tildeTyrangle$,
beginalign
langle Tx_n,yrangle &= langle x_n,tildeTyranglerightarrow_nrightarrowinftylangle x,tildeTyrangle=langle Tx,yrangle.
endalign

Therefore, $Tx_nrightarrow Tx$ as $nrightarrowinfty$.



Third Attempt:



Assume $|x_n-x|rightarrow 0$ as $nrightarrowinfty$. Then,
beginalign
langle Tx_n-Tx,Tx_n-Txrangle=langle x_n-x,x_n-xrangle=|x_n-x|^2.
endalign



By assumption $|x_n-x|^2rightarrow 0$ as $nrightarrowinfty$. Hence,
beginalign
langle Tx_n-Tx,Tx_n-Txrangle = |Tx_n-Tx|^2rightarrow 0text as nrightarrowinfty.
endalign

Therefore, $T$ is continuous.










share|cite|improve this question











$endgroup$







  • 2




    $begingroup$
    The last inequality basically implies that the norm of T is bounded or that it is continuous
    $endgroup$
    – Andres Mejia
    4 hours ago






  • 1




    $begingroup$
    Comment on the second attempt: you showed that $Tx_n to Tx$ weakly, not in norm. Off-topic comment: I admire your tenacity. Keep trying!
    $endgroup$
    – Umberto P.
    4 hours ago











  • $begingroup$
    Thank you. Do you have a hint?
    $endgroup$
    – Jack
    4 hours ago










  • $begingroup$
    Third attempt made. Although not sure if this holds either.
    $endgroup$
    – Jack
    3 hours ago













2












2








2


1



$begingroup$



Note: Please do not give a solution; I am curious to understand why my solution is incorrect, and would prefer guidance to help me complete the question myself. Thank you.





Let $mathcalH$ be a Hilbert space, and suppose that $TintextHom(mathcalH,mathcalH)$. Suppose that there exists an operator $tildeT:mathcalHrightarrowmathcalH$ such that,
beginalign
langle Tx,yrangle =langle x,tildeTyrangle,
endalign

$forall x,yinmathcalH$. Show that $T$ is continuous.



My current solution is as follows:



Assume for all $delta>0$ there exists $n>NinmathbbN$ such that,
beginalign
|x_n-x|<delta.
endalign

Then,
beginalign
langle Tx_n-Tx,Tx_n-Txrangle &= |Tx_n-Tx|^2\
&leq|Tx_n-Tx|=|T(x_n-x)|\
&leq|T||x_n-x|rightarrow 0text as nrightarrowinfty.
endalign



What am I doing wrong? I notice I do not use the existence of $tildeT$.



Second Attempt:



Assume $langle x_n,xrangle rightarrow langle x,xrangle$ as $nrightarrowinfty$. Then, given $langle Tx,yrangle = langle x,tildeTyrangle$,
beginalign
langle Tx_n,yrangle &= langle x_n,tildeTyranglerightarrow_nrightarrowinftylangle x,tildeTyrangle=langle Tx,yrangle.
endalign

Therefore, $Tx_nrightarrow Tx$ as $nrightarrowinfty$.



Third Attempt:



Assume $|x_n-x|rightarrow 0$ as $nrightarrowinfty$. Then,
beginalign
langle Tx_n-Tx,Tx_n-Txrangle=langle x_n-x,x_n-xrangle=|x_n-x|^2.
endalign



By assumption $|x_n-x|^2rightarrow 0$ as $nrightarrowinfty$. Hence,
beginalign
langle Tx_n-Tx,Tx_n-Txrangle = |Tx_n-Tx|^2rightarrow 0text as nrightarrowinfty.
endalign

Therefore, $T$ is continuous.










share|cite|improve this question











$endgroup$





Note: Please do not give a solution; I am curious to understand why my solution is incorrect, and would prefer guidance to help me complete the question myself. Thank you.





Let $mathcalH$ be a Hilbert space, and suppose that $TintextHom(mathcalH,mathcalH)$. Suppose that there exists an operator $tildeT:mathcalHrightarrowmathcalH$ such that,
beginalign
langle Tx,yrangle =langle x,tildeTyrangle,
endalign

$forall x,yinmathcalH$. Show that $T$ is continuous.



My current solution is as follows:



Assume for all $delta>0$ there exists $n>NinmathbbN$ such that,
beginalign
|x_n-x|<delta.
endalign

Then,
beginalign
langle Tx_n-Tx,Tx_n-Txrangle &= |Tx_n-Tx|^2\
&leq|Tx_n-Tx|=|T(x_n-x)|\
&leq|T||x_n-x|rightarrow 0text as nrightarrowinfty.
endalign



What am I doing wrong? I notice I do not use the existence of $tildeT$.



Second Attempt:



Assume $langle x_n,xrangle rightarrow langle x,xrangle$ as $nrightarrowinfty$. Then, given $langle Tx,yrangle = langle x,tildeTyrangle$,
beginalign
langle Tx_n,yrangle &= langle x_n,tildeTyranglerightarrow_nrightarrowinftylangle x,tildeTyrangle=langle Tx,yrangle.
endalign

Therefore, $Tx_nrightarrow Tx$ as $nrightarrowinfty$.



Third Attempt:



Assume $|x_n-x|rightarrow 0$ as $nrightarrowinfty$. Then,
beginalign
langle Tx_n-Tx,Tx_n-Txrangle=langle x_n-x,x_n-xrangle=|x_n-x|^2.
endalign



By assumption $|x_n-x|^2rightarrow 0$ as $nrightarrowinfty$. Hence,
beginalign
langle Tx_n-Tx,Tx_n-Txrangle = |Tx_n-Tx|^2rightarrow 0text as nrightarrowinfty.
endalign

Therefore, $T$ is continuous.







functional-analysis continuity hilbert-spaces






share|cite|improve this question















share|cite|improve this question













share|cite|improve this question




share|cite|improve this question








edited 3 hours ago







Jack

















asked 5 hours ago









JackJack

887




887







  • 2




    $begingroup$
    The last inequality basically implies that the norm of T is bounded or that it is continuous
    $endgroup$
    – Andres Mejia
    4 hours ago






  • 1




    $begingroup$
    Comment on the second attempt: you showed that $Tx_n to Tx$ weakly, not in norm. Off-topic comment: I admire your tenacity. Keep trying!
    $endgroup$
    – Umberto P.
    4 hours ago











  • $begingroup$
    Thank you. Do you have a hint?
    $endgroup$
    – Jack
    4 hours ago










  • $begingroup$
    Third attempt made. Although not sure if this holds either.
    $endgroup$
    – Jack
    3 hours ago












  • 2




    $begingroup$
    The last inequality basically implies that the norm of T is bounded or that it is continuous
    $endgroup$
    – Andres Mejia
    4 hours ago






  • 1




    $begingroup$
    Comment on the second attempt: you showed that $Tx_n to Tx$ weakly, not in norm. Off-topic comment: I admire your tenacity. Keep trying!
    $endgroup$
    – Umberto P.
    4 hours ago











  • $begingroup$
    Thank you. Do you have a hint?
    $endgroup$
    – Jack
    4 hours ago










  • $begingroup$
    Third attempt made. Although not sure if this holds either.
    $endgroup$
    – Jack
    3 hours ago







2




2




$begingroup$
The last inequality basically implies that the norm of T is bounded or that it is continuous
$endgroup$
– Andres Mejia
4 hours ago




$begingroup$
The last inequality basically implies that the norm of T is bounded or that it is continuous
$endgroup$
– Andres Mejia
4 hours ago




1




1




$begingroup$
Comment on the second attempt: you showed that $Tx_n to Tx$ weakly, not in norm. Off-topic comment: I admire your tenacity. Keep trying!
$endgroup$
– Umberto P.
4 hours ago





$begingroup$
Comment on the second attempt: you showed that $Tx_n to Tx$ weakly, not in norm. Off-topic comment: I admire your tenacity. Keep trying!
$endgroup$
– Umberto P.
4 hours ago













$begingroup$
Thank you. Do you have a hint?
$endgroup$
– Jack
4 hours ago




$begingroup$
Thank you. Do you have a hint?
$endgroup$
– Jack
4 hours ago












$begingroup$
Third attempt made. Although not sure if this holds either.
$endgroup$
– Jack
3 hours ago




$begingroup$
Third attempt made. Although not sure if this holds either.
$endgroup$
– Jack
3 hours ago










1 Answer
1






active

oldest

votes


















4












$begingroup$

The problem is that we can't assume that $T$ has a finite norm. Before we add that condition about having an adjoint map $tildeT$, we're simply assuming that $T$ is a linear map.



In fact, a linear map between normed vector spaces is continuous if and only if it has a finite operator norm. You assumed the statement we were trying to prove.



Second attempt: The assumption here should have been that $x_nto x$, as in the others. Then, yes, $langle Tx_n,yrangle to langle Tx,yrangle$ for each $y$. This is real progress. But, as stated in the comments, it's weak convergence rather than convergence in norm. Not quite there.



Third attempt: No, $langle Tu,Turangle$ is not equal to $langle u,urangle$ - it's equal to $langle u,tildeTTurangle$, and you don't know what $tildeTT$ does. This is not helpful.



All, right, lets go back to the attempt that made some progress. Are you familiar with the uniform boundedness principle? One consequence of that theorem is that any sequence of points in a Hilbert space that converges weakly is bounded. Can we use this to ensure that $T$ is a bounded operator?






share|cite|improve this answer











$endgroup$












  • $begingroup$
    So is the idea for me to use $tildeT$ to cancel out the operator norm in my final inequality?
    $endgroup$
    – Jack
    4 hours ago










  • $begingroup$
    @Jack no, that won't rescue the proof.
    $endgroup$
    – Umberto P.
    4 hours ago










  • $begingroup$
    New proof attempt. Please check if you can.
    $endgroup$
    – Jack
    4 hours ago










Your Answer





StackExchange.ifUsing("editor", function ()
return StackExchange.using("mathjaxEditing", function ()
StackExchange.MarkdownEditor.creationCallbacks.add(function (editor, postfix)
StackExchange.mathjaxEditing.prepareWmdForMathJax(editor, postfix, [["$", "$"], ["\\(","\\)"]]);
);
);
, "mathjax-editing");

StackExchange.ready(function()
var channelOptions =
tags: "".split(" "),
id: "69"
;
initTagRenderer("".split(" "), "".split(" "), channelOptions);

StackExchange.using("externalEditor", function()
// Have to fire editor after snippets, if snippets enabled
if (StackExchange.settings.snippets.snippetsEnabled)
StackExchange.using("snippets", function()
createEditor();
);

else
createEditor();

);

function createEditor()
StackExchange.prepareEditor(
heartbeatType: 'answer',
autoActivateHeartbeat: false,
convertImagesToLinks: true,
noModals: true,
showLowRepImageUploadWarning: true,
reputationToPostImages: 10,
bindNavPrevention: true,
postfix: "",
imageUploader:
brandingHtml: "Powered by u003ca class="icon-imgur-white" href="https://imgur.com/"u003eu003c/au003e",
contentPolicyHtml: "User contributions licensed under u003ca href="https://creativecommons.org/licenses/by-sa/3.0/"u003ecc by-sa 3.0 with attribution requiredu003c/au003e u003ca href="https://stackoverflow.com/legal/content-policy"u003e(content policy)u003c/au003e",
allowUrls: true
,
noCode: true, onDemand: true,
discardSelector: ".discard-answer"
,immediatelyShowMarkdownHelp:true
);



);













draft saved

draft discarded


















StackExchange.ready(
function ()
StackExchange.openid.initPostLogin('.new-post-login', 'https%3a%2f%2fmath.stackexchange.com%2fquestions%2f3149977%2fcontinuity-of-linear-operator-between-hilbert-spaces%23new-answer', 'question_page');

);

Post as a guest















Required, but never shown

























1 Answer
1






active

oldest

votes








1 Answer
1






active

oldest

votes









active

oldest

votes






active

oldest

votes









4












$begingroup$

The problem is that we can't assume that $T$ has a finite norm. Before we add that condition about having an adjoint map $tildeT$, we're simply assuming that $T$ is a linear map.



In fact, a linear map between normed vector spaces is continuous if and only if it has a finite operator norm. You assumed the statement we were trying to prove.



Second attempt: The assumption here should have been that $x_nto x$, as in the others. Then, yes, $langle Tx_n,yrangle to langle Tx,yrangle$ for each $y$. This is real progress. But, as stated in the comments, it's weak convergence rather than convergence in norm. Not quite there.



Third attempt: No, $langle Tu,Turangle$ is not equal to $langle u,urangle$ - it's equal to $langle u,tildeTTurangle$, and you don't know what $tildeTT$ does. This is not helpful.



All, right, lets go back to the attempt that made some progress. Are you familiar with the uniform boundedness principle? One consequence of that theorem is that any sequence of points in a Hilbert space that converges weakly is bounded. Can we use this to ensure that $T$ is a bounded operator?






share|cite|improve this answer











$endgroup$












  • $begingroup$
    So is the idea for me to use $tildeT$ to cancel out the operator norm in my final inequality?
    $endgroup$
    – Jack
    4 hours ago










  • $begingroup$
    @Jack no, that won't rescue the proof.
    $endgroup$
    – Umberto P.
    4 hours ago










  • $begingroup$
    New proof attempt. Please check if you can.
    $endgroup$
    – Jack
    4 hours ago















4












$begingroup$

The problem is that we can't assume that $T$ has a finite norm. Before we add that condition about having an adjoint map $tildeT$, we're simply assuming that $T$ is a linear map.



In fact, a linear map between normed vector spaces is continuous if and only if it has a finite operator norm. You assumed the statement we were trying to prove.



Second attempt: The assumption here should have been that $x_nto x$, as in the others. Then, yes, $langle Tx_n,yrangle to langle Tx,yrangle$ for each $y$. This is real progress. But, as stated in the comments, it's weak convergence rather than convergence in norm. Not quite there.



Third attempt: No, $langle Tu,Turangle$ is not equal to $langle u,urangle$ - it's equal to $langle u,tildeTTurangle$, and you don't know what $tildeTT$ does. This is not helpful.



All, right, lets go back to the attempt that made some progress. Are you familiar with the uniform boundedness principle? One consequence of that theorem is that any sequence of points in a Hilbert space that converges weakly is bounded. Can we use this to ensure that $T$ is a bounded operator?






share|cite|improve this answer











$endgroup$












  • $begingroup$
    So is the idea for me to use $tildeT$ to cancel out the operator norm in my final inequality?
    $endgroup$
    – Jack
    4 hours ago










  • $begingroup$
    @Jack no, that won't rescue the proof.
    $endgroup$
    – Umberto P.
    4 hours ago










  • $begingroup$
    New proof attempt. Please check if you can.
    $endgroup$
    – Jack
    4 hours ago













4












4








4





$begingroup$

The problem is that we can't assume that $T$ has a finite norm. Before we add that condition about having an adjoint map $tildeT$, we're simply assuming that $T$ is a linear map.



In fact, a linear map between normed vector spaces is continuous if and only if it has a finite operator norm. You assumed the statement we were trying to prove.



Second attempt: The assumption here should have been that $x_nto x$, as in the others. Then, yes, $langle Tx_n,yrangle to langle Tx,yrangle$ for each $y$. This is real progress. But, as stated in the comments, it's weak convergence rather than convergence in norm. Not quite there.



Third attempt: No, $langle Tu,Turangle$ is not equal to $langle u,urangle$ - it's equal to $langle u,tildeTTurangle$, and you don't know what $tildeTT$ does. This is not helpful.



All, right, lets go back to the attempt that made some progress. Are you familiar with the uniform boundedness principle? One consequence of that theorem is that any sequence of points in a Hilbert space that converges weakly is bounded. Can we use this to ensure that $T$ is a bounded operator?






share|cite|improve this answer











$endgroup$



The problem is that we can't assume that $T$ has a finite norm. Before we add that condition about having an adjoint map $tildeT$, we're simply assuming that $T$ is a linear map.



In fact, a linear map between normed vector spaces is continuous if and only if it has a finite operator norm. You assumed the statement we were trying to prove.



Second attempt: The assumption here should have been that $x_nto x$, as in the others. Then, yes, $langle Tx_n,yrangle to langle Tx,yrangle$ for each $y$. This is real progress. But, as stated in the comments, it's weak convergence rather than convergence in norm. Not quite there.



Third attempt: No, $langle Tu,Turangle$ is not equal to $langle u,urangle$ - it's equal to $langle u,tildeTTurangle$, and you don't know what $tildeTT$ does. This is not helpful.



All, right, lets go back to the attempt that made some progress. Are you familiar with the uniform boundedness principle? One consequence of that theorem is that any sequence of points in a Hilbert space that converges weakly is bounded. Can we use this to ensure that $T$ is a bounded operator?







share|cite|improve this answer














share|cite|improve this answer



share|cite|improve this answer








edited 2 hours ago

























answered 4 hours ago









jmerryjmerry

14.3k1629




14.3k1629











  • $begingroup$
    So is the idea for me to use $tildeT$ to cancel out the operator norm in my final inequality?
    $endgroup$
    – Jack
    4 hours ago










  • $begingroup$
    @Jack no, that won't rescue the proof.
    $endgroup$
    – Umberto P.
    4 hours ago










  • $begingroup$
    New proof attempt. Please check if you can.
    $endgroup$
    – Jack
    4 hours ago
















  • $begingroup$
    So is the idea for me to use $tildeT$ to cancel out the operator norm in my final inequality?
    $endgroup$
    – Jack
    4 hours ago










  • $begingroup$
    @Jack no, that won't rescue the proof.
    $endgroup$
    – Umberto P.
    4 hours ago










  • $begingroup$
    New proof attempt. Please check if you can.
    $endgroup$
    – Jack
    4 hours ago















$begingroup$
So is the idea for me to use $tildeT$ to cancel out the operator norm in my final inequality?
$endgroup$
– Jack
4 hours ago




$begingroup$
So is the idea for me to use $tildeT$ to cancel out the operator norm in my final inequality?
$endgroup$
– Jack
4 hours ago












$begingroup$
@Jack no, that won't rescue the proof.
$endgroup$
– Umberto P.
4 hours ago




$begingroup$
@Jack no, that won't rescue the proof.
$endgroup$
– Umberto P.
4 hours ago












$begingroup$
New proof attempt. Please check if you can.
$endgroup$
– Jack
4 hours ago




$begingroup$
New proof attempt. Please check if you can.
$endgroup$
– Jack
4 hours ago

















draft saved

draft discarded
















































Thanks for contributing an answer to Mathematics Stack Exchange!


  • Please be sure to answer the question. Provide details and share your research!

But avoid


  • Asking for help, clarification, or responding to other answers.

  • Making statements based on opinion; back them up with references or personal experience.

Use MathJax to format equations. MathJax reference.


To learn more, see our tips on writing great answers.




draft saved


draft discarded














StackExchange.ready(
function ()
StackExchange.openid.initPostLogin('.new-post-login', 'https%3a%2f%2fmath.stackexchange.com%2fquestions%2f3149977%2fcontinuity-of-linear-operator-between-hilbert-spaces%23new-answer', 'question_page');

);

Post as a guest















Required, but never shown





















































Required, but never shown














Required, but never shown












Required, but never shown







Required, but never shown

































Required, but never shown














Required, but never shown












Required, but never shown







Required, but never shown







Popular posts from this blog

Magento 2 - Add success message with knockout Planned maintenance scheduled April 23, 2019 at 23:30 UTC (7:30pm US/Eastern) Announcing the arrival of Valued Associate #679: Cesar Manara Unicorn Meta Zoo #1: Why another podcast?Success / Error message on ajax request$.widget is not a function when loading a homepage after add custom jQuery on custom themeHow can bind jQuery to current document in Magento 2 When template load by ajaxRedirect page using plugin in Magento 2Magento 2 - Update quantity and totals of cart page without page reload?Magento 2: Quote data not loaded on knockout checkoutMagento 2 : I need to change add to cart success message after adding product into cart through pluginMagento 2.2.5 How to add additional products to cart from new checkout step?Magento 2 Add error/success message with knockoutCan't validate Post Code on checkout page

Fil:Tokke komm.svg

Where did Arya get these scars? Unicorn Meta Zoo #1: Why another podcast? Announcing the arrival of Valued Associate #679: Cesar Manara Favourite questions and answers from the 1st quarter of 2019Why did Arya refuse to end it?Has the pronunciation of Arya Stark's name changed?Has Arya forgiven people?Why did Arya Stark lose her vision?Why can Arya still use the faces?Has the Narrow Sea become narrower?Does Arya Stark know how to make poisons outside of the House of Black and White?Why did Nymeria leave Arya?Why did Arya not kill the Lannister soldiers she encountered in the Riverlands?What is the current canonical age of Sansa, Bran and Arya Stark?